Practice Questions Exam 2

¡Supera tus tareas y exámenes ahora con Quizwiz!

Women with an intact uterus should be treated with both estrogen and progestin due to: A. Increased risk for endometrial cancer if estrogen alone is used B. Combination therapy provides the best relief of menopausal vasomotor symptoms C. Reduced risk for colon cancer with combined therapy D. Lower risk of developing blood clots with combined therapy

A. Increased risk for endometrial cancer if estrogen alone is used

Which side effect can be caused by statin therapy? A. Fatty liver B. Flushing C. New-onset DM D. Gout

C. New-onset DM

Which of the following drugs is most appropriate for managing urge incontinence? A. Imipramine B. Estrogens C. Phenylpropanolamine D. Oxybutynin E. Bethanechol

D. Oxybutynin

Which arrhythmia can be treated with lidocaine? A. Paroxysmal supraventricular tachycardia B. Atrial fibrillation C. Atrial flutter D. Ventricular tachycardia

D. Ventricular tachycardia Lidocaine has little effect on atrial or AV nodal tissue. Thus, it is used for ventricular arrhythmias such as ventricular tachycardia.

Which of the following agents has the lowest incidence of dry mouth as a side effect according to its package insert? A. Trospium IR B. Solifenacin C. Fesoterodine D. Oxybutynin XL E. Oxybutynin TDS

E. Oxybutynin TDS

A 50-year-old woman is newly diagnosed with overactive bladder with urge incontinence. Her current medications include clarithromycin and fluoxetine. Which of the following agents is most likely to provide symptom control with the fewest drug interactions? A. Bethanechol B. Darifenacin C. Estrogen D. Solifenacin E. Trospium

E. Trospium

Podagra

Gout in the big toe

Patients prescribed this drug long-term may develop osteoporsis

Heparin

Short half-life; prescribed as constant infusion for therapeutic anticoagulation

Heparin

Standard prophylaxis dose is 5,000 units SubQ q8h

Heparin

Stop the infusion to reverse effects (usually sufficient)

Heparin

May precipitate acute gout attack, do not prescribe in these patients

Niacin

Potential to decrease LDL as much as 25%

Niacin

Slowly titrate dose to a maximum of 2 grams daily (ER form)

Niacin

Has minimal effects on heart rate or blood pressure

Ranolazine

May prolong the QT interval

Ranolazine

Usually reserved for angina refractory to other medications

Ranolazine

Very Expensive

Ranolazine

R.R. has an AUA Symptom Score of 6. A diagnosis of mild LUTS due to BPH is made. Which treatment of choice is recommended? A. Watchful waiting B. Adrenergic antagonist C. 5-α reductase inhibitor D. Anticholinergic agent E. Minimally invasive surgery

A. Watchful waiting

Which one of the following drug-induced incontinence pairs is correct? A. Terazosin/functional incontinence B. Enalapril/stress incontinence C. Estradiol/overflow incontinence D. Tolterodine/urge incontinence E. Morphine/urge incontinence

B. Enalapril/stress incontinence

Which of the following is correct regarding the pharmacokinetics of the bisphosphonates? A. Bisphosphonates are well absorbed after oral administration. B. Food or other medications greatly impair absorption of bisphosphonates. C. Bisphosphonates are mainly metabolized via the cytochrome P450 systems. D. Elimination half-life of bisphosphonates ranges from 4 to 6 hours.

B. Food or other medications greatly impair absorption of bisphosphonates. Food and other medications do decrease absorption of bisphosphonates, which are already poorly absorbed (less than 1%) after oral administration. Bisphosphonates are cleared from the plasma by binding to bone and being cleared by the kidney (not metabolized by the CYP-450 system). The elimination half-life may be years.

May mask the signs of hypoglycemia in diabetic patients

Beta Blockers

Preferred treatment for prevention of recurrent ischemia (with past MI)

Beta Blockers

Should avoid in patients with poorly controlled asthma

Beta Blockers

Which medication should be prescribed to all anginal patients to treat an acute attack? A. Isosorbide dinitrate B. Nitroglycerin patch C. Nitroglycerin sublingual tablet or spray D. Ranolazine

C. Nitroglycerin sublingual tablet or spray The other options will not provide prompt relief of angina and should not be used to treat an acute attack.

All of the following are risk factors for stress incontinence except: A. Traumatic childbirth B. Radical prostatectomy C. Surgery for benign prostatic hyperplasia D. Nasal decongestant for cold symptoms E. Menopause

D. Nasal decongestant for cold symptoms

May worsen or precipitate gall bladder disease

Fibrates

Also known as "statins"

HMG-CoA Reductase Inhibitors

Available in oral and transdermal forms

Long-acting nitrates

Do not use as monotherapy for prevention of recurrent ischemia

Long-acting nitrates

Example: Isosorbide mononitrate

Long-acting nitrates

Nitrate used for prevention of recurrent ischemia

Long-acting nitrates

Requires nitrate-free interval

Long-acting nitrates

Uric Acid

Produced from the natural breakdown of your body's cells and from the foods you eat. Most of the uric acid is filtered out by the kidneys and passes out of the body in urine. High levels of uric acid in the blood can cause solid crystals to form within joints. This causes a painful condition called gout.

Contraindicated with strong CYP3A4 inducers or inhibitors

Ranolazine

Uricosuric

Substances that increase the excretion of uric acid in the urine, thus reducing the concentration of uric acid in the blood plasma. In general, this effect is achieved by action on the proximal tubule of the kidney.

Tophus

A deposit of uric acid crystals in people with longstanding hyperuricemia (high levels of uric acid in the blood)

O.P. is a 65-year-old female who has been diagnosed with postmenopausal osteoporosis. She has no history of fractures and no other pertinent medical conditions. Which of the following would be most appropriate for management of her osteoporosis? A. Alendronate B. Calcitonin C. Denosumab D. Raloxilene E. Teriparatide

A. Alendronate Bisphosphonates are first-line therapy for osteoporosis in postmenopausal women without contraindications. Calcitonin and raloxilene are alternatives, but may be less efficacious (especially for nonvertebral fractures). Teriparatide and denosumab should be reserved for patients at high risk of those who fail other therapies.

In a follow-up visit after 6 months, your patient states that he is still having occasional flare-ups of podagra every few weeks, despite his best efforts at controlling his diet and losing 20 lbs. His recent lab tests indicate a serum urate level of 6.7 mg/dL, which is consistent with his lab values 6 months ago. What medication would you recommend that could reduce his rate of uric acid production, and consequently reduce his incidence of gout attacks? A. Allopurinol B. Etanercept C. Hydroxychloroquine D. Probenecid

A. Allopurinol Allopurinol is a xanthine oxidase inhibitor, and will reduce the rate of uric acid production associated with purine metabolism. It is effective as a chronic treatment for gout Etanercept is an "anti-TNF" drug that is given by s.c. injection, and is used in the treatment of rheumatoid arthritis, but not gout. It does not affect uric acid production. This drug is used in the treatment of rheumatoid arthritis, and malaria, but not gout. It does not affect uric acid production. Probenecid does not affect "the rate of uric acid production." It does promote the loss of uric acid by the kidney, and is indicated primarily for patients who can't tolerate allopurinol, or require additional uric acid lowering.

All of the following are adverse effects of amiodarone except which of the following? A. Cinchonism B. Hypothyroidism C. Hyperthyroidism D. Pulmonary fibrosis E. Blue skin discoloration

A. Cinchonism Cinchonism is a constellation of symptoms (blurred vision, tinnitus, headache, psychosis) that is known to occur with quinidine. All other options are adverse effects with amiodarone that require close monitoring.

A recently married 38-year-old woman who recently recovered from a pulmonary embolism is seeking a prescription for contraception. Which of the following would be contraindicated? A. Combination (estrogen + progestin) oral contraceptive pills. B. Depot medroxyprogestrone acetate C. Implanon implant D. Levonorgestrel releasing IUD E Progestin-only mini pill.

A. Combination (estrogen + progestin) oral contraceptive pills.

Estrogen replacement therapy in menopausal women: A. Decreases bone loss accompanying osteoporosis B. Requires higher doses of estrogen than with oral contraceptive therapy C. May cause atrophic vaginitis D. May induce hot flashes

A. Decreases bone loss accompanying osteoporosis

Delayed access to toileting facilities due to severe arthritic pain is a risk factor for which of the following? A. Functional incontinence B. Urge incontinence C. Bladder overactivity D. Bladder underactivity E. Both B and C are correct

A. Functional incontinence

Which of the following conditions may be responsible for fecal impaction in an elderly patient? A. Functional incontinence B. Urge incontinence C. Bladder overactivity D. Bladder underactivity E. None of the above

A. Functional incontinence

Question 06/06 D.D. is a 50-year-old male with newly diagnosed hypertension. His comorbidities include diabetes and chronic hepatitis C infection with moderate liver impairment. He requires two drugs for initial treatment of his hypertension. Which should be prescribed in combination with a thiazide diuretic? A. Lisinopril B. Spironolactone C. Fosinopril D. Furosemide E. Hydralazine

A. Lisinopril Because D.D. has diabetes, he has a compelling indication for an ACE inhibitor or ARB for the treatment of his hypertension and prevention of diabetic nephropathy. However, most ACE inhibitors undergo hepatic conversion to active metabolites, so his hepatic impairment is of concern. Because lisinopril is one of the two ACE inhibitors that does not undergo hepatic conversion to active metabolites, it is the best choice. Fosinopril is the only ACE inhibitor that is not eliminated primarily by the kidneys, but does undergo hepatic conversion. An additional diuretic like spironolactone or furosemide is not indicated. D.D. does not have a compelling indication for hydralazine.

Which one of the following is the most appropriate treatment approach in a patient at risk for developing OA? A. Promote lifestyle changes targeted at risk factors for OA B. Initiate tramadol 50 mg every 4 to 6 hours as needed. C. Initiate ibuprofen 200 mg every 6 hours as needed. D. Initiate acetaminophen 325 mg every 4 to 6 hours as needed, and promote lifestyle changes targeted at risk factors for OA. E. Initiate ibuprofen 200 mg every 6 hours as needed, and promote lifestyle changes targeted at risk factors for OA

A. Promote lifestyle changes targeted at risk factors for OA

A 62-year-old patient with a history of asthma and vasospastic angina states that he gets chest pain both with exertion and at rest, about ten times per week. One sublingual nitroglycerin tablet always relieves his symptoms, but this medication gives him an awful headache every time he takes it. Which is the best option for improving his angina? A. Change to sublingual nitroglycerin spray B. Add amlodipine C. Add propanolol D. Replace nitroglycerin with ranolazine

B. Add amlodipine Calcium channel blockers are preferred for vasospastic angina. Beta-blockers can actually worsen vasospastic angina; furthermore, nonselective beta-blockers should be avoided in patients with asthma. The nitroglycerin spray would also be expected to cause headache, so this is not the best choice. Ranolazine is not indicated for immediate relief of an angina attack, nor is it a first-line option.

A patient diagnosed with metabolic syndrome and a congenital heart defect was taking atorvastatin 20 mg daily. His primary care physician recently increased the dose of atorvastatin to 40 mg daily. However, the patient complains of muscle pain and weakness shortly after the dose is escalated. Based on this information and the results below, what would be the most appropriate intervention? Non-HDL cholesterol: 118 mg/dL (3.05 mmol/L) LDL cholesterol: 78 mg/dL (2.02 mmol/L) Triglycerides: 198 mg/dL (2.24 mmol/L) HDL cholesterol: 39 mg/dL (1.01 mmol/L) Total cholesterol: 157 mg/dL (4.06 mmol/L) A. Continue current therapy and monitor his progress B. Add ezetimibe 10 mg daily C. Add niacin ER 500 mg at bedtime D. Switch to pravastatin 20 mg at bedtime

B. Add ezetimibe 10 mg daily

A 64 year old man with a history of angina of effort is scheduled for elective coronary angioplasty (PCI) and placement of a coronary stent. Associated with the procedure he is given eptifibatide. What is the mechanism of action of this agent? A. ADP receptor inhibitor B. Blockade of GP IIB/IIIA receptors C. Cox inhibitor D. Phosphodiesterase inhibitor E. Protease-activated receptor-1 antagonist

B. Blockade of GP IIB/IIIA receptors Antiplatelet drugs with this mechanism of action (abciximab, eptifibatide, and tirofiban) are indicated as adjunct antiplatelet therapy for PCI and treatment of acute coronary syndrome.

Absolute contraindications to estrogen therapy include which of the following? A. History of any type of cancer B. Clotting disorders C. History of tension headaches D. Orthostatic hypertension

B. Clotting disorders

During a return visit, your patient states that the combination of NSAIDs, dietary restrictions and moderate weight loss have helped control his symptoms of gout. However taking ibuprofen has been causing him stomach pain, and he has been reluctant to take the higher doses needed to control his toe pain. What other anti-inflammatory drug could you recommend for acute therapy that doesn't have similar side effects on the stomach? A. Allopurinol B. Colchicine C. Keterolac D. Methylprednisolone

B. Colchicine Colchicine works by inhibiting microtubules involved in the chemotaxis of neutrophils. It is a good drug of 2nd choice for treating gout, especially when NSAIDs are contraindicated. Allopurinol is not an anti-inflammatory drug and will not treat this patient's pain. Keotrolac is prescription only NSAID which could worsen his GI issues. Steroids can cause side effects similar to NSAIDs on the stomach.

A 56-year-old man presents to the emergency room with complaints of swelling, redness, and pain in his right leg. The patient is diagnosed with acute DVT and requires treatment with an anticoagulant. All of the following are approved for treatment of this patient's DVT except which of the following drugs? A. Rivaroxaban B. Dabigatran C. Enoxaparin D. Heparin

B. Dabigatran Dabigatran is only approved for the prevention of stroke in nonvalvular atrial fibrillation; it is not approved for the treatment of acute DVT. All of the other options are approved for treatment of acute DVT.

T.T. is a 55-year-old female who has been diagnosed with postmenopausal osteoporosis. She has a past medical history of ethanol abuse, alcoholic liver disease, erosive esophagitis, and hypothyroidism. Which of the following would be the primary reason oral bisphosphonates should be used with caution in this patient? A. Age B. Erosive esophagitis C. Liver disease D. Thyroid disease

B. Erosive esophagitis Bisphosphonates are known to cause esophageal irritation and should be used with caution in a patient with a history of erosive esophagitis. Age is not a factor for consideration in bisphosphonate use. Liver disease is not a contraindication to bisphosphonate use, since bisphosphonates are mainly cleared via the kidney. Thyroid disease is not a contraindication to bisphosphonate use, although overaggressive replacement of thyroid may contribute to osteoporosis.

Which of the following is CORRECT regarding finasteride? A. Finasteride is associated with significant hypotension. B. Finasteride is associated with birth defects. C. Finasteride is effective within 2 weeks of initiation. D. Finasteride is renally eliminated.

B. Finasteride is associated with birth defects. Because finasteride inhibits the conversion of testosterone to its active form, it may cause significant developmental defects in the male genitalia of a developing fetus. As such, it is contraindicated in pregnancy. Unlike the α-blockers, the 5-α reductase inhibitors are not associated with hypotension. Finasteride may take up to 12 months before it is effective. Finally, finasteride is metabolized via CYP450 and is not renally eliminated.

Which of the following should be avoided in a patient with a history of severe anaphylactic reaction to sulfa medications? A. Amiloride B. Hydrochlorothiazide C. Mannitol D. Spironolactone E. Triamterene

B. Hydrochlorothiazide Hydrochlorothiazide, like many thiazide and thiazide-like diuretics, contains a sulta moiety within its chemical structure. It is important to avoid use in those individuals with severe hypersensitivity to sulfa medications. It may be used with caution, however, in those with only minor reaction to sulfa medications.

A 67-year-old man (5'4", 280 lbs.) complains of significant pain associated with the region immediately behind his left big toe. His toe appears red and swollen, consistent with podagra. When asked about his diet, he admits to eating shrimp several times a week, and drinking 3-4 beers with dinner most evenings. Recently, he has been eating steak every other evening following a successful deer hunting trip. His blood tests indicate a moderately high level of uric acid (7 mg/dL). In addition to recommended weight loss and changes in diet, which of the following would be the first choice treatment for his condition? A. Acetaminophen B. Ibuprofen C. Methotrexate D. Pegloticase

B. Ibuprofen As a NSAID, its an affordable OTC choice for pain relief that is commonly effective in gout. NSAIDs do not, however, treat the underlying cause for gout. Weight loss, dietary restrictions, and/or additional types of drug therapy may be needed to reduce the incidence of gout reoccurrence. Acetaminophen and methotrexate are not used for gout treatment. Pegloticase is used for urate lowering therapy, but is not a good drug of first choice. It has to be given by intravenous injection, and has a black box warning as well

A 21-year-old recently married woman with a history of cigarette smoking is seeking contraceptive therapy. When discussing her long-term plans, she indicates that she does not wish to become pregnant for at least the next 5 years. After discussing the available therapeutic options, you both decide that the best choice for her current needs is a reversible form of contraception that is highly efficacious, and also has a very long duration of action (>1 year). Which therapeutic option for contraception seems most appropriate for this patient? A. Combination oral contraceptive pills. B. Implanon implant C. Medroxyprogestrone s.c. injection D. Progestin-only mini pills E. Transdermal contraceptive patch F. Tubal ligation

B. Implanon implant

A 23-year-old woman is prescribed a progestin-only mini pill for contraception. Which advice would you consider most appropriate before writing her prescription? A. Taking these pills for more than a year in duration frequently results in sterility. B. It is imperative that progestin mini pills be taken at the same time each day. C. These pills can help with acne. D. These pills are potent suppressors of ovulation, and have a 2-week grace period before pregnancy is likely to occur if a pill is missed.

B. It is imperative that progestin mini pills be taken at the same time each day.

A 53-year-old woman has severe vasomotor symptoms (hot flashes) associated with menopause. She has no pertinent past medical history or surgical history. Which of the following would be most appropriate for her? A. Conjugated estrogens vaginal cream B. Oral estradiol and medroxyprogesterone C. Estradiol transdermal patch D. Injectable medroxyprogesterone

B. Oral estradiol and medroxyprogesterone

Which of the following statements is CORRECT regarding the mechanism of action of phosphodiesterase-5 (PDE-5) inhibitors? A. PDE-5 inhibitors increase prostaglandin production B. PDE-5 inhibitors enhance the effect of nitric oxide C. PDE-5 inhibitors cause vasoconstriction of the erection chamber D. PDE-5 inhibitors antagonize cyclic GMP

B. PDE-5 inhibitors enhance the effect of nitric oxide PDE-5 inhibitors enhance the effect of nitric oxide by preventing the breakdown of cGMP. PDE-5 inhibitors do not affect prostaglandin production. Although blood is drawn to the erection chamber, PDS-5 inhibitors allow for this via vasodilation, not vasoconstriction. PDE-5 inhibitors prevent the breakdown of cGMP, but do not antagonize its action.

A 52-year-old postmenopausal, overweight woman complains of small volumes of urine leakage when she coughs, laughs, or practices yoga. She denies urinary frequency or incontinence at night. Which of the following conditions is most likely the cause of her incontinence? A. Functional incontinence B. Stress incontinence C. Overactive bladder D. Overflow incontinence E. Urge incontinence

B. Stress incontinence

An 80-year-old male is taking warfarin indefinitely for the prevention of deep venous thrombosis. He is a compliant patient with a stable INR and has no issues with bleeding or bruising. He is diagnosed with a urinary tract infection and is prescribed sulfamethoxazole/trimethoprim. What effect will this have on his warfarin therapy? A. Sulfamethoxazole/trimethoprim will decrease the anticoagulant effect of warfarin B. Sulfamethoxazole/trimethoprim will increase the anticoagulant effect of warfarin. C. Sulfamethoxazole/trimethoprim will activate platelet activity D. Sulfamethoxazole/trimethoprim will not change anti-coagulation status.

B. Sulfamethoxazole/trimethoprim will increase the anticoagulant effect of warfarin. Sulfamethoxazole/trimethoprim has a significant drug interaction with warfarin, such that it will inhibit warfarin metabolism. Therefore, sulfamethoxazole/trimethoprim will cause increased anti-coagulation, and the patient will need to have his warfarin dose decreased and INR checked frequently while he is on this antibiotic.

Which of the following is an important difference between terazosin and tamsulosin? A. Terazosin blocks the α1A receptors, while tamsulosin blocks α1A and α1B receptors. B. Terazosin blocks α1A and α1B receptors, while tamsulosin blocks α1A receptors. C. Terazosin blocks 5-α reductase, while tamsulosin blocks PDE-5. D. Terazosin must be taken with food, while tamsulosin can be taken on an empty stomach.

B. Terazosin blocks α1A and α1B receptors, while tamsulosin blocks α1A receptors. Tamsulosin is more selective for the α1A receptor, found in the prostrate. Terazosin blocks α1A; however, terazosin also blocks α1B. Neither one blocks 5-α reductase nor PDE-5. Tamsulosin should be taken with food, while terazosin does not need to be taken with food.

A patient who arrives in the emergency department suffering from chest pain associated with apparent unstable angina is given a drug that reduces the risk of thrombosis by inhibiting the formation of thromboxane A2. Which drug was given? A. apiximab B. aspirin C. cangrelor D. clopidogrel E. heparin

B. aspirin Aspirin irreversibly inhibits platelet cyclooxygenase, which results in decreased production of thromboxane A2.

Abrupt withdrawal may be life threatening

Beta Blockers

Which one of the following regimens is considered an adequate trial of acetaminophen before assessing treatment success or failure? A. 325 mg every 6 hours for 1 month B. 325 mg every 6 hours for 2 months C. 1000 mg every 6 hours for 1 month D. 500 mg every 8 hours for 2 months E. 325 mg every 4 to 6 hours for 1 month

C. 1000 mg every 6 hours for 1 month

For which of the following patients would concomitant naproxen and a PPI be most appropriate? A. 49-year-old woman with metabolic syndrome B. 70-year-old man with a history of Helicobacter pylori peptic ulcer disease C. 58-year-old man with a history of myocardial infarction and previous gastric ulcer with ibuprofen use D. 67-year-old man with a history of GI bleed from previous indomethacin use E. 70-year-old woman with chronic heart failure

C. 58-year-old man with a history of myocardial infarction and previous gastric ulcer with ibuprofen use

Which must heparin bind to in order to exert its anticoagulant effect? A. GP IIb/IIIa receptor B. Thrombin C. Antithrombin III D. von Willebrand factor

C. Antithrombin III Heparin binds to antithrombin III, causing a conformational change. This heparin/antithrombin III complex then inactivates thrombin and factor Xa.

Kristine would like to start HRT to treat the significant vasomotor symptoms she is experiencing during menopause. Education for a woman considering hormone replacement would include which of the following? A. Explaining that HRT is totally safe if used short term B. Telling her to ignore media hype regarding HRT C. Discussing the advantages and risks of HRT D. Encouraging the patient to use phytoestrogens with the HRT

C. Discussing the advantages and risks of HRT

A patient is worried about starting terazosin because he is very sensitive to side effects of medications. Which of the following adverse effects would be most expected in this patient? A. Erectile dysfunction B. Gynecomastia C. Dizziness D. Vomiting

C. Dizziness Because of the α-blocking properties, terazosin commonly causes dizziness (this may be related to orthostatic hypotension). ED and gynecomastia would be unexpected with α-blockers. While mot any drug may cause nausea and vomiting, terazosin is much more likely to cause dizziness.

Which one of the following side effects of finasteride could be treated with a phosphodiesterase inhibitor? A. Gynecomastia B. Decrease libido C. Erectile dysfunction D. Retrograde ejaculation E. Hirustism

C. Erectile dysfunction

Which of the following statements is true regarding the use of glucosamine/chondroitin for OA? A. Treatment benefit is moderate but consistent among all types and severities of OA. B. Because glucosamine/chondroitin is regulated as a dietary supplement, product standards are consistent and reliable. C. Glucosamine/chondroitin is generally well tolerated, but treatment benefits are uncertain. D. Both glucosamine and chondroitin pose a high risk for anaphylaxis in patients with shellfish allergy. E. Glucosamine-containing products are contraindicated in patients with uncontrolled disease

C. Glucosamine/chondroitin is generally well tolerated, but treatment benefits are uncertain.

All of the following medications can be useful for managing stable angina in a patient with coronary artery disease except which of the following? A. Amlodipine B. Atenolol C. Immediate-release nifedipine D. Isosorbide dinitrate

C. Immediate-release nifedipine The short-acting dihydropyridine calcium channel blocker nifedipine should be avoided in CAD patients as this can worsen angina; however, the extended-release formulation can be used.

Question 05/06 Which is the appropriate choice for hypertension treatment during pregnancy? A. Aliskiren B. Fosinopril C. Labetalol D. Valsartan

C. Labetalol Labetolol, a beta blocker, is an appropriate choice for a hypertensive pregnant patient. ACE inhibitors, ARBs, and the direct renin inhibitor, aliskiren, are all contraindicated in pregnancy due to their potential for fetal harm.

When discussing contraceptive options with your young patient, you also discuss the effectiveness of different methods with regards to protection from sexually transmitted infections. Among the options listed below, which would be MOST effective for this purpose? A. Combination (estrogen + progestin) oral contraceptive pills. B. Intramuscular medroxyprogesterone acetate C. Male or female condoms D. Spermicide E. Tubal ligation

C. Male or female condoms

What is the clinical term for angina caused by coronary vasospasm? A. Classic Angina B. Myocardial Infarction C. Prinzmetal Angina D. Unstable Angina

C. Prinzmetal Prinzmetal angina is angina caused by vasospasm of the coronary arteries. It is also known as vasospastic or variant angina. The other answers refer to angina (with varying levels of severity) caused by atherosclerosis.

A 65-year-old male experiences uncontrolled angina attacks that limit his ability to do household chores. He is adherent to a maximized dose of beta-blocker with a low heart rate and low blood pressure. He was unable to tolerate an increase in isosorbide mononitrate due to headache. Which is the most appropriate addition to his antianginal therapy? A. Amlodipine B. Aspirin C. Ranolazine D. Verapamil

C. Ranolazine Ranolazine is the best answer. The patient's blood pressure is low, so verapamil and amlodipine may drop blood pressure further. Verapamil may also decrease heart rate. Ranolazine can be used when other agents are maximized, especially when blood pressure is well controlled. The patient will need a baseline ECG and lab work to ensure safe use of this medication.

V.S. is a 70-year-old female who is being started on ibandronate once monthly for the treatment of osteoporosis. Which of the following is important to communicate to this patient? A. Take this medication with orange juice to increase absorption. B. Take this medication after meals to minimize stomach upset. C. Remain upright for at least 60 minutes after taking this medication. D. Adverse effects may include blood clots and leg cramps.

C. Remain upright for at least 60 minutes after taking this medication. Patients need to remain upright for 60 minutes after ibandronate (30 minutes for other bisphosphonates). Ibandronate should be given on an empty stomach with plain water only. Bisphosphonates, unlike raloxilene, are not associated with blood clots and leg cramps.

A 22-year-old woman who has been sexually active for several years comes to your clinic to discuss contraceptive options. Before prescribing a medication, what type of blood test should be performed first? A. Activin B. GnRH C. hCG D. Inhibin

C. hCG

Decrease heart rate AND increase oxygen supply

Calcium Channel Blockers

Preferred for vasospastic angina

Calcium Channel Blockers

When used with Beta Blockers, may increase risk for severe bradycardia or AV block

Calcium Channel Blockers

Therapy with celecoxib, rather than a non-selective NSAID, would be most appropriate for which of the following patients? A. 58-year-old woman with metabolic syndrome and gastroesophageal reflux disease B. 69-year-old man with a history of Helicobacter pylori peptic ulcer disease C. 53-year-old woman with a strong family history of myocardial infarction and previous GI upset with ibuprofen use D. 47-year-old man with a history of GI bleed from previous indomethacin use E. 70-year-old woman with chronic renal insufficiency who takes low-dose aspirin for cardioprotection

D. 47-year-old man with a history of GI bleed from previous indomethacin use

Ongoing monitoring for women on ERT includes which of the following? A. Lipid levels, repeated annually if abnormal B. Annual health history and review of risk profile C. Annual mammogram D. All of the above

D. All of the above

The advantage of vaginal estrogen preparations in the treatment of vulvovaginal atrophy and dryness is: A. Ability to deliver higher doses of estrogen in a non-oral form B. The vaginal cream formula provides moisture to the vaginal area C. Relief of symptoms without increasing cardiovascular risk D. All of the above

D. All of the above

Which of the following BEST describes the mechanism of action of alprostadil? A. Alprostadil blocks cGMP B. Alprostadil blocks nitric acid. C. Alprostadil increase PDE-5. D. Alprostadil increases cAMP.

D. Alprostadil increases cAMP. Through an unknown mechanism, alprostadil (a synthetic prostaglandin) increases levels of cAMP, causing smooth muscle relaxation. Alprostadil does not affect cGMP, nitric oxide, or PDE-5.

Compared with oral NSAIDs, topical NSAIDs are: A. More effective for hip OA B. More effective for knee OA C. Less effective for hand OA D. Associated with fewer systemic toxicities E. All of the above

D. Associated with fewer systemic toxicities

Which of the following is the best initial choice for preventing bedwetting in a child who plans to go for a sleepover at a friend's house? A. Oxybutynin B. Imipramine C. Vasopressin D. Desmopressin E. Flavoxate

D. Desmopressin

Which of the following is correct regarding digoxin when used for atrial fibrillation? A. Digoxin works by blocking voltage-sensitive calcium channels. B. Digoxin is used for rhythm control in patients with atrial fibrillation. C. Digoxin increases conduction velocity through the AV node. D. Digoxin levels of 1 to 2 ng/mL are desirable in the treatment of atrial fibrillation.

D. Digoxin levels of 1 to 2 ng/mL are desirable in the treatment of atrial fibrillation. Digoxin works by inhibiting the Na+/K+-ATPase pump. It decreases conduction velocity through the AV node and is used for rate control in atrial fibrillation *not rhythm control). Digoxin levels between 1 and 2 ng/mL are more likely to exhibit negative chronotropic effects desired in atrial fibrillation or flutter. A serum drug concentration between 0.5 and 0.8 ng/mL is for symptomatic management of heart failure.

Which of the following distinguishes the PDE-5 inhibitor tadalafil from avanafil, vardenafil, and sildenafil? A. Efficacy B. Safety C. Speed of onset D. Duration of action E. All of the above

D. Duration of action

Which of the following treatments for pediatric enuresis has the lowest relapse rate after an adequate duration of trial? A. Desmopressin B. Fluid restriction C. Imipramine D. Enuresis alarm therapy E. Awakening from sleep before enuresis episodes

D. Enuresis alarm therapy

Which of the following is the first-line therapy for a motivated child with primary monosymptomatic enuresis? A. Oxybutynin B. Imipramine C. Desmopressin D. Enuresis alarms E. Flavoxate

D. Enuresis alarms

Which formulation of testosterone replacement is associated with wide swings of serum testosterone concentrations? A. Daily transdermal testosterone gel B. Daily transdermal testosterone patch C. Twice daily buccal testosterone D. Every two week intramuscular (IM) testosterone cypionate

D. Every two week intramuscular (IM) testosterone cypionate

Which type of therapy would be most appropriate for this patient? 58-year-old woman with Type 2 DM Evidence of end-organ damage Total cholesterol 210 mg/dL (5.43 mmol/L) HDL cholesterol 45 mg/dL (1.16 mmol/L) Triglycerides 850 mg/dL (9.61 mmol/L) Non-HDL cholesterol 165 mg/dL (4.27 mmol/L) A. Atorvastatin 10 mg daily B. Pravastatin 20 mg daily C. Cholestyramine one scoop twice daily D. Gemfibrozil 600 mg twice daily E. Alcohol and carbohydrate restriction

D. Gemfibrozil 600 mg twice daily

Compared to nonselective NSAIDs, COX-2—selective agents: A. Reduce OA-related pain to a greater extent B. Have a lower risk for cardiovascular adverse events C. Have a lower risk for adverse renal effects D. Have a lower risk of gastrointestinal bleeds E. All of the above

D. Have a lower risk of gastrointestinal bleeds

Use of an α-adrenergic antagonist or a 5α-reductase inhibitor can be expected to have which of the following clinical effects in patients with symptomatic BPH? A. Decrease peak urinary flow rate B. Increase post void residual urine volume C. Increase detrusor relaxation D. Improve bladder emptying E. Increase urinary frequency

D. Improve bladder emptying

A patient returns to her health care provider for routine monitoring 3 months after her hypertension regimen was modified. Labs reveal elevated serum potassium. Which is likely responsible for this hyperkalemia? A. Chlorthalidone B. Clonidine C. Furosemide D. Losartan E. Nifedpine

D. Losartan Losartan, an ARB, can cause an increase in serum potassium similar to ACE inhibitors. Furosemide and chlorthalidone can cause a decrease in serum potassium. Nifedipine and clonidine do not affect potassium levels.

A 72-year-old male presents to the primary care clinic complaining of chest tightness and pressure that is increasing in severity and frequency. His current medications include atenolol, lisinopril, and nitroglycerin. Which intervention is most appropriate at this time? A. Add amlodipine B. Initiate isosorbide mononitrate C. Initiate ranolazine D. Refer the patient to the nearest emergency room for evaluation.

D. Refer the patient to the nearest emergency room for evaluation Crescendo angina is indicative of unstable angina that requires further workup.

A male patient is placed on a new medication and notes that his breasts have become enlarged and tender to the touch. Which medication is he most likely taking? A. Chlorthalidone B. Furosemide C. Hydrochlorothiazide D. Spironolactone E. Triamterene

D. Spironolactone An adverse drug reaction to spironolactone is gynecomastia due to its effects on androgens and progesterone in the body. Eplerenone may be a suitable alternative if the patient is in need of an aldosterone antagonist but has a history of gynecomastia.

Which of the following is the BEST description of the mechanism of action of terazosin? A. Terazosin blocks 5-α reductase B. Terazosin blocks α1A reductase C. Terazosin blocks PDE-5 D. Terazosin blocks both α1A and α1B receptors.

D. Terazosin blocks both α1A and α1B receptors. Terazosin blocks both α1A and α1B receptors. Terazosin does not affect 5-α reductase or PDE-5.

Which one of the following drug regimens is most appropriate for a patient with OA who has with chronic renal insufficiency (creatinine clearance 28 mL/min [0.47 mL/s]) and who has failed acetaminophen monotherapy? A. Nabumetone 50 mg twice daily B. Celecoxib 100 mg twice daily C. Naproxen 500 mg twice daily D. Tramadol 100 mg three times daily E. Oxycodone 5 mg immediate-release every 4 to 6 hours as needed

D. Tramadol 100 mg three times daily

A 38-year-old woman with a 20-year history of cigarette smoking (1/2 pack per day) is seeking a prescription for contraception. Which of the following would have the greatest contraindication for use in this patient? A. Depot medroxyprogestrone B. Implanon implant C. Levonorgestrel releasing IUD D. Transdermal patch

D. Transdermal patch

A 70-year-old female is diagnosed with nonvalvular atrial fibrillation. Her past medical history is significant for chronic kidney disease, and her renal function is moderately diminished. All of the following anticoagulants would be expected to require a reduced dosage in this patient except which of the following? A. Apixaban B. Dabigatran C. Rivaroxaban D. Warfarin

D. Warfarin Warfarin does not require dosage adjustment in renal dysfunction. The INR is monitored and dosage adjustments are made on the basis of this information. All of the other agents are renally cleared to some extent and require dosage adjustments in renal dysfunction.

A 38-year-old female patient has been diagnosed with hypercholesterolemia. You are prescribing atorvastatin (Lipitor®) 20 mg PO qhs to be started after lab results are obtained. Which laboratory result would be of most concern? A. ALT 57 IU/L B. Creatinine 1.4 mg/dL C. CRP 3 mg/dL D. hCG 287 IU/L

D. hCG 287 IU/L

A 59 year old man who recently survived a myocardial infarction is on drug therapy designed to reduce his cardiovascular risk for a second MI. During the course of his therapy, he is accidentally given an overdose of unfractionated heparin. What treatment would be most effective in counteracting the effects of this overdose? A. deferoxamine B. dipryidamidole C. idarucizumab D. protamine sulfate E. vitamin k

D. protamine sulfate Protamine sulfate is a very basic compound that chelates heparin, which is highly acidic.

These oral medications have a quick onset

Direct Acting Oral Anticoagulants

Nonpharmacologic therapies for OA include which of the following? A. Stretching B. Application of heat C. Weight loss D. Occupational therapy E. All of the above

E. All of the above

Patients attempting treatment with topical capsaicin should be counseled on which of the following? A. Do not allow contact with eyes or mucous membranes B. When applying to knee, wash hands after application C. May take 2 weeks of daily treatment to experience benefit D. May experience burning sensation at application site E. All of the above

E. All of the above

Which of the following agents is most appropriate for managing atonic bladder? A. Imipramine B. Estrogens C. Phenylpropanolamine D. Oxybutynin E. Bethanechol

E. Bethanechol

In an older male with diagnosed symptomatic benign prostatic hyperplasia, addition of which of the following may lead to acute urinary retention? A. Pseudoephedrine B. Prazosin C. Bethanechol D. Oxybutynin E. Both A and D are correct

E. Both A and D are correct

Stimulation of muscarinic cholinergic receptors in the bladder muscle may be responsible for: A. Functional incontinence B. Urge incontinence C. Bladder overactivity D. Bladder underactivity E. Both B and C are correct.

E. Both B and C are correct.

Which of the following statements is true regarding pediatric enuresis? A. A relapse is defined by more than 3 wet nights per week after a period of dryness. B. Enuresis alarm therapy is not helpful when reinitiated in a child with a relapse of enuresis. C. If a child fails to remain dry for 2 consecutive nights, he shall be penalized by removing rewards previously gained for agreed-upon behaviors. D. Initial management with education, motivational and behavioral therapy should be discontinued after 2 weeks of trial. E. Children should drink no more than 8 oz (240 mL) of fluid 1 hour before to 8 hours after taking oral desmopressin.

E. Children should drink no more than 8 oz (240 mL) of fluid 1 hour before to 8 hours after taking oral desmopressin.

Which of the following is true regarding intraarticular corticosteroid use in a patient achieving a partial response to naproxen 500 mg twice daily for left knee OA? A. Pain relief will occur rapidly and persist for up to 6 months. B. Intraarticular corticosteroids should not be used concomitantly with naproxen. C. Intraarticular corticosteroids are useful for polyarticular symptoms due to their systemic mechanism of action. D. The affected joint can be injected monthly until response. E. Intraarticular corticosteroids may be more effective if an inflammatory component is present.

E. Intraarticular corticosteroids may be more effective if an inflammatory component is present.

It has been estimated that almost 82% of pregnancies in women between the ages of 15-19 years old are unintended. When discussing contraceptive options with your 19-year-old patient, you compare and contrast the least and most effective methods available. Among the options listed below, which is considered the LEAST effective as typically used? A. Combined oral contraceptive pill B. Copper IUD C. Depot medroxyprogestrone D. Etonogestrel implant E. Male condom

E. Male condom

A 78-year-old woman has been newly diagnosed with atrial fibrillation. She is not currently having symptoms of palpitations or fatigue. Which is appropriate to initiate for rate control as an outpatient? A. Amiodarone B. Dronedarone C. Esmolol D. Flecainide E. Metoprolol

E. Metoprolol Only Esmolol and Metoprolol are options to control rate. The other options are used for rhythm control in patients with atrial fibrillation. Since esmolol is IV only, the only option to start as an outpatient is metoprolol.

Your patient's gout continues to flare up occasionally, despite treatment. You are considering prescribing a "uricosuric" drug that can be taken along with his other medications for preventing gout flare-ups. This drug acts by inhibiting anionic transporters in the proximal tubule, which reduces the renal reabsorption of uric acid by by 30-40%. On the downside, it can also promote the formation of uric acid kidney stones in patients with high urinary uric acid levels and is not very effective in patients with reduced renal function. Which drug best fits this description? A. Anakinra B. Azathioprine C. Colchicine D. Febuxostat E. Probenecid

E. Probenecid Probenecid inhibits URAT1, an anionic transporter expressed on the apical (urinary side) of renal tubules that plays a major role in uric acid reabsorption. Anakinra, colchicine and febuxostat can be used for gout, but none are uricosuric drugs. Azathioprine is an immunosuppressant prodrug that is metabolized to mercaptopurine. It interferes with purine metabolism involved in lymphoid cell proliferation. It is used in the management of severe arthritis and for preventing rejection of renal transplants.

Which diuretic has been shown to improve blood pressure in resistant hypertension or those already treated with three blood pressure medications including a thiazide or thiazide-like diuretic? A. Chlorthalidone B. Indapamide C. Furosemide D. Mannitol E. Spironolactone

E. Spironolactone Resistant hypertension, defined by the use of three or more medications without reaching the blood pressure goal, often responds well to aldosterone antagonists. This effect can be seen in those with or without elevated aldosterone levels.

A 65 year old man suffering from chest pain and difficulty breathing for the past 20 minutes is brought to the emergency department of a small rural community hospital. He is given a chewable aspirin while his history is taken. His ECG indicates ST segment elevation (STEMI), and his pO2 is 85%. He is given supplemental oxygen while an intravenous line is inserted. Which of the following would be the best choice to improve blood flow to the ischemic region of the heart if angioplasty (PCI) is not immediately available? A. clopidogrel B. dabigatran C. enoxaparin D. abciximab E. reteplase

E. reteplase Reteplase is a plasminogen activator that selectively binds to fibrin in a thrombus and converts the entrapped plasminogen to plasmin, which then digests the fibrin clot.

Block biliary reabsorption of dietary cholesterol

Ezetemibe

Decrease LDL as much as 18%

Ezetemibe

Maximum effect seen in approximately 2 weeks

Ezetemibe

Often prescribed with a statin as trials for monotherapy were not conducted

Ezetemibe

Very well tolerated relative to other medications in class

Ezetemibe

Increase the effects of warfarin which increases bleeding risk

Fibrates

Maximum effect seen in 2-4 weeks (depending on which drug in class is used)

Fibrates

Prescribed in the setting of high triglycerides and low LDL

Fibrates

1st line treatment choice for dyslipidemia

HMG-CoA Reductase Inhibitors

Have potential to decrease LDL as much as 66%

HMG-CoA Reductase Inhibitors

Inhibits conversion of HMG CoA reductase to L-mevalonic acid (and cholesterol)

HMG-CoA Reductase Inhibitors

Maximum effect in 4-6 weeks; follow-up no sooner than 1-2 months

HMG-CoA Reductase Inhibitors

May increase risk of new-onset DM

HMG-CoA Reductase Inhibitors

Newest treatment guidelines recommend dosing by "intensity"

HMG-CoA Reductase Inhibitors

Can reverse effects 1 mg of protamine for every 100 units of this drug

Heparin

Dose adjustments are made based on aPTT

Heparin

Enhances activity of AT3 neutralizing BOTH factors IIa and Xa

Heparin

Goal to get aPTT into range of 1.5-2.5 times control within 24 hours

Heparin

Almost always prescribed by SubQ

Low Molecular Weight Heparin (LMWH)

Anti-Xa levels not routinely monitored, except special populations like peds

Low Molecular Weight Heparin (LMWH)

Drug of choice for VTE treatment or prophylaxis for pregnant patients

Low Molecular Weight Heparin (LMWH)

Enhances the activity ot AT3, neutralizing mostly Xa and to a much lesser extent

Low Molecular Weight Heparin (LMWH)

HIT is a contraindication for heparin use, as all as for the use of this drug

Low Molecular Weight Heparin (LMWH)

Protamine sulfate can be used for reversal; complete reversal may not happen

Low Molecular Weight Heparin (LMWH)

Provider should prescribe prefilled syringes whenever possible

Low Molecular Weight Heparin (LMWH)

Renal function assessment is critical for this type of heparin

Low Molecular Weight Heparin (LMWH)

Also known as vitamin B3, a nutritional supplement

Niacin

Available as IR, SR, and ER formulation

Niacin

Dose-related facial flushing may occur

Niacin

Highly effective for triglyceride reduction AND can also lower LDL

Niacin

Maximum effect seen in 3-5 weeks

Niacin

CYP2C19 metabolizes this drug; must assess for DIs involving this isoenzyme

Warfarin

Decreased production of vitamin K dependent clotting factors (2,7,9,10)

Warfarin

Diet and drug interactions are a major consideration when prescribing this drug

Warfarin

Effects of this drug can be reversed with phytonadione (vitamin K)

Warfarin

Like the DOACs, this drug is only available orally

Warfarin

Loading doses are no longer recommended for this drug

Warfarin

Onset of anticoagulant effect is usually 2-7 days

Warfarin

PT-INR is used to guide dose adjustments

Warfarin

This drug is contraindicated during pregnancy (category X)

Warfarin


Conjuntos de estudio relacionados

KNH 213 Community Nutrition Questions

View Set

Chapter 6: Choosing Your Courses and Academic Program (Notes)

View Set

Unit 2 Barron's: Interactions Among the Branches of Government

View Set

Ch. 16 health assessment -Assessing eyes

View Set

Chapter Exam Life Premiums and Benefits

View Set

SCM Chapter 10: Global Logistics & International Trade

View Set